S AT # 1 1 Re a d i n g E x p l a n a ti o n s (fu l l , v ... · a rri ve a t t h e f a mi l y’...

43
SAT #11 Reading Explanations (full, v.1) Text by Paragraph / Things to Notice somebody arrives to the ink shop noticing everything in the shop, and its description father opens up the ink and ledger a memory of precious auntie and her advice back from the memory to the ink stick pride in her father’s work some names the setting, the scene, the process the attitude and tone (reverential, etc.) 1.R.11 (hard) ****lots of wrong things, it is the character visiting the shop and not old widow lau who is the customer*** Answer Choice D is the correct answer because the Passage depicts Old Widow Lau arriving to the family’s ink shop and the narrator observing her and her father, taking interest in the ink and her father’s words and concluding, “I was very proud to hear Father speak of our family’s ink this way.” This most closely matches Answer Choice D which says, “A character’s visit to her family’s ink shop deepens her appreciation of her family’s work.” Answer Choice A is incorrect because, although the character does remember her Precious Auntie during the visit, this memory doesn’t become the primary focus of the Passage. Answer Choice B is incorrect because, although there is a small reunion between the narrator and Big Uncle and Little Uncle, this reunion doesn’t take center stage in the Passage. Answer Choice C is incorrect because the Passage doesn’t talk about the narrator’s father’s interests. How to solve this? The Question asks, “Which choice best summarizes the passage?” Because this is a [Big Picture] Question, we should try to have a general understanding of the Passage first, and then try to identify and specific sentences which frame the Passage as a whole. [For the basic breakdown, see above]. Looking for specific texts, we can look at the introductory blurb as well as the introductory paragraph. This Passage doesn’t offer too much in either, which is more typical of literary passages, except for setting up the scene of the Old Widow Lau’s arrival to the ink shop. The concluding sentence, however, says, “I was very proud to hear Father speak of our family’s ink this way,” which does provide something of a summation for the Passage. At this point, we should go through the answer choices for the question, focusing on looking for errors within them.

Transcript of S AT # 1 1 Re a d i n g E x p l a n a ti o n s (fu l l , v ... · a rri ve a t t h e f a mi l y’...

SAT #11 Reading Explanations (full, v.1)

Text by Paragraph / Things to Notice

● somebody arrives to the ink shop ● noticing everything in the shop, and its description ● father opens up the ink and ledger ● a memory of precious auntie and her advice ● back from the memory to the ink stick ● pride in her father’s work

● some names ● the setting, the scene, the process ● the attitude and tone (reverential, etc.)

1.R.11 (hard) ****lots of wrong things, it is the character visiting the shop and not old widow lau who is the customer*** Answer Choice D is the correct answer because the Passage depicts Old Widow Lau arriving to the family’s ink shop and the narrator observing her and her father, taking interest in the ink and her father’s words and concluding, “I was very proud to hear Father speak of our family’s ink this way.” This most closely matches Answer Choice D which says, “A character’s visit to her family’s ink shop deepens her appreciation of her family’s work.” Answer Choice A is incorrect because, although the character does remember her Precious Auntie during the visit, this memory doesn’t become the primary focus of the Passage. Answer Choice B is incorrect because, although there is a small reunion between the narrator and Big Uncle and Little Uncle, this reunion doesn’t take center stage in the Passage. Answer Choice C is incorrect because the Passage doesn’t talk about the narrator’s father’s interests. How to solve this? The Question asks, “Which choice best summarizes the passage?” Because this is a [Big Picture] Question, we should try to have a general understanding of the Passage first, and then try to identify and specific sentences which frame the Passage as a whole. [For the basic breakdown, see above]. Looking for specific texts, we can look at the introductory blurb as well as the introductory paragraph. This Passage doesn’t offer too much in either, which is more typical of literary passages, except for setting up the scene of the Old Widow Lau’s arrival to the ink shop. The concluding sentence, however, says, “I was very proud to hear Father speak of our family’s ink this way,” which does provide something of a summation for the Passage. At this point, we should go through the answer choices for the question, focusing on looking for errors within them.

Answer Choice A says, “A character’s arrival at her family’s ink shop sparks fond memories of her favorite aunt.” The first part of this answer choice is correct, because Old Widow Lau does arrive at the family’s ink shop. The second part might also seem correct, because Lines 46-59 show the narrator remembering her Precious Auntie, so we can keep this option. Answer Choice B says that “a character’s surprise visit leads to a happy reunion at her family’s ink shop.” This answer choice might also seem correct at first, because it describes a visitor coming to the shop like the answer choices, and because something like a reunion occurs in the first paragraph, when the narrator reunites with Big and Little Uncle. However, we should note that Old Widow Lau’s arrival is never presented as a “surprise visit,” and that the reunion between the Uncles and the narrators doesn’t have a primary place in the Passage, so we can eliminate this option. Answer Choice C says that “a character comes to understand her father’s ambitions while visiting her family’s ink shop.” Because the Passage doesn’t discuss her father’s ambitions in the Passage, we should eliminate this option. Answer Choice D says that “a character’s visit to her family’s ink shop deepens her appreciate of her family’s work.” Because the Passage does end with a mention of the narrator’s appreciation for her family work following Old Widow Lau’s visit, we should keep this option. That leaves us with Answer Choice A and Answer Choice D. Because the beginnings of these answers choices are identical, we should focus on the second part and decide if the Passage sparks fond memories of the narrator’s favorite aunt or deepens her appreciation of her family’s work. To solve this, we should note that although Paragraph 4 does show the narrator remembering her Precious Auntie, the Passage doesn’t make this its primary focus and presents it as an aside. In Lines 60-64, the Passage transitions away from this memory when the narrator says, “I remembered this, and yet that day in the ink shop, I listened to what Father was saying, and his words became far more important than anything Precious Auntie had thought.” After this transition, the Passage ends with, “I was very proud to hear Father speak of our family’s ink this way,” giving us a more central summary of the Passage. Thus, we should choose Answer Choice A for our correct answer. [Plus, a more general answer choice]. 2.R.11 Answer Choice B is the correct answer because the Passage does present the theme that “quality is achieved through deliberate effort,” when, for example, in Lines 50-51, the narrator remember Precious Auntie saying, “You can never be an artist if your work comes without effort.” Answer Choice A is incorrect because, although the Passage does present family relationships, it never expressly focuses on the need to nurture these. Answer Choice C is incorrect because the Passage doesn’t discuss how hard work leads to material compensation (and because of rhetorical flair). Answer Choice D is incorrect because the Passage also doesn’t discuss how “creativity needs to be expressed concretely.”

How to solve this? The Question asks, “A main theme of the passage is that [...]?” To solve this, we should first have a general understanding and memory of the Passage, and then work through the answer choices, finding errors and eliminating those options. Answer Choice A says that “family relationships should be nurtured.” Either by memory, or by searching through the Passage, we can look for evidence in the Passage for this idea. After doing so, we should note that no such evidence exists. Answer Choice B says that “quality is achieved through deliberate effort,” which is supported by Lines 50-51 and subsequent lines in the paragraph, so we should keep this option. Going through Answer Choices C and D, we should also look for supporting evidence either by memory or by scanning, and note that it doesn’t exist, so we should eliminate these options. That leaves Answer Choice B which we should choose as our correct answer. 3.R.11 Answer Choice B is the correct answer because the Passage shows us multiple instances where the narrator shows that she’s “attuned to her immediate surroundings.” For example, in Lines 22-23 she says, “I tried to notice everything so I could later tell GaoLing what I had seen [...],” and in Lines 63-64, “‘Look here,’ Father said to his customer, and I looked.” Answer Choices A, B and C are incorrect because no evidence in the Passage shows the narrator being “reserved around unfamiliar people,” “sympathetic to the needs of others,” or “anxious about her responsibilities.” How to solve this? The Question asks, “Throughout the passage, the narrator is portrayed as someone who is [...]?” To solve this, we should just work with our basic memory of the Passage and the narrator’s portrayal in it, and then focus on eliminating options and finding support for others. Answer Choice A says that the narrator is portrayed as someone who is, “reserved around unfamiliar people.” We should recognize this option as unlikely because it has some [pejorative flair] and makes a strong claim that we should expect to have noticed. Moreover, if we go back to Passage, we find no evidence in support of this. Answer Choice B says that the narrator is portrayed as someone who is “attuned to her immediate surroundings.” Because this answer choice is neutral in both meaning and tone, and because sufficient evidence for this portrayal of the narrator runs throughout the Passage, we should keep this option. Answer Choice C says that the narrator is “sympathetic to the needs of others.” This answer choice is also fairly neutral, and therefore plausible. But, looking back over the Passage, we should note that no evidence exists for it, so we should eliminate this option. Answer Choice D says that the narrator is portrayed as someone who is, “anxious about her responsibilities.” This answer choice is both “loud” and somewhat critical, so we should consider it unlikely. Moreover, we should note that no evidence exists for the answer choice so we should eliminate this option.

4.R.11 & 5.R.11 Answer Choices A and C are the correct answers because Lines 15-18 say, “Old Widow Lau refused their invitation three times, exclaiming that my father and uncles must be too busy for visitors. She made weak efforts to leave,” which most closely matches Answer Choice A, that Old Widow Lau’s reluctance was “feigned, because she is not genuinely firm in her resolve.” For Question 5, Answer Choices A, B and D are all incorrect because neither give us evidence about Old Widow Lau’s reluctance to stay for tea. For Question 4, Answer Choices B and D are incorrect because the Passage doesn’t tell us that the family has been planning the visit or that Old Widow Lau is exhausted. Answer Choice C is incorrect, because the store only has one customer and isn’t described as being unusually busy. How to solve this? Question 4 asks, “It can be most reasonably inferred from the passage that Old Widow Lau’s reluctance to stay for tea is [...]?” Because this is a Paired Question, to solve it we should first go through the answer choices for Question 5, looking for any Text that tells us about Old Widow Lau’s reluctance to stay for tea. Because this is an earlier question in the series of questions, we should expect our lines to come earlier in the Passage and, [using the Question Order Rule], before Lines 46. But since all of the texts fit these criteria, this won’t help us too much. For Question 5, Answer Choice A says, “At last, Old Widow Lau was done haggling with the driver and we stepped inside Father’s shop. It was north-facing, quite dim inside, and perhaps this was why Father did not see us at first.” Because this answer choice doesn’t tell us anything about the Old Widow Lau’s reluctance to take tea, we should eliminate this option. Answer Choice B says, “So I called his name in a shy voice. And he squinted at me, then laughed and announced our arrival to Little Uncle, who apologized many times for not rushing over sooner to greet us.” This answer choice also doesn’t tell us about her reluctance, so we should eliminate this answer choice. Answer Choice C says, “Old Widow Lau refused their invitation three times, exclaiming that my father and uncles must be too busy for visitors. She made weak efforts to leave.” Because this answer choice does tell us about her reluctance to take tea, we should keep this option. Answer Choice D says, “Then Little Uncle brought us hot tea and sweet oranges, as well as bamboo latticework fans with which to cool ourselves.” Because this option comes after Old Widow Lau’s initial reluctance and shows her actually sitting down for tea, we should eliminate this option. This leaves us with Answer Choice C, which we should choose for our correct answer. Lines 15-18 say, “Old Widow Lau refused their invitation three times, exclaiming that my father and uncles must be too busy for visitors. She made weak efforts to leave.” We should recognize that her exclamation that the “father and uncles must be too busy” doesn’t match her “weak efforts to leave,” and therefore that her reluctance is not entirely genuine. So, we should expect our answer choice to relate to this.

For Question 4, Answer Choice A says that her reluctance was, “feigned, because she is not genuinely firm in her resolve.” This does match our understanding of the Text, that Old Widow Lau didn’t mean what she said. Answer Choice B says that her reluctance was, “inconsiderate, because the family has been planning her visit.” Because this answer choice has some [pejorative flair], we shouldn’t expect it to be our answer. Moreover, the Passage gives us no evidence that the family “has been planning her visit,” nor does our specific Text present her as being inconsiderate, so we should eliminate this option. Answer Choice C says that her reluctance is “appropriate, because the shop is unusually busy.” Because the Passage doesn’t tell us that the shop is especially busy, but only that it has one customer, we should eliminate this option. Answer Choice D says that her reluctance is “ill-advised, because she is exhausted from the journey.” Because the Text doesn’t tell us that Old Widow Lau is exhausted, we should eliminate this option. That leaves only Answer Choice A, which we should choose as our correct answer. 6.R.11 Answer Choice A is the correct answer because Paragraph 2 describes the physical details of the father’s shop, focusing specifically on the glass display cases, of which the narrator makes the comment, “They looked so much nicer than they had in the ink-making studio at Immortal Heart village.” This most closely matches Answer Choice A, which says that, compared to the ink-making studio at Immortal Heart village, the narrator’s family ink shop “displays the family’s ink more impressively.” Answer Choices B and C are incorrect because the Passage doesn’t describe the family’s ink shop as being “more conveniently located for the public” or as providing “greater individual attention to customers.” Answer Choice D is incorrect because the narrator comments on the quality and appearance of the display, not on the size of the space. How to solve this? The Question asks, “The narrator indicates that the contrast between the ink-making studio at Immortal Heart village and her family’s ink shop is that the ink shop [...]?” To solve this, we should first go back to the Passage and look for what the narrator says about her family’s ink shop. The [Question Order Rule] tells us that we should expect our Text to come before Lines 46, and we can begin scanning in that area for “Immortal Heart Village.” Lines 30-32 say, “They looked so much nicer than they had in the ink-making studio at Immortal Heart village.” Checking our references, we should note that the “they” mentioned here refers to the glass display cases that the narrator admires. Thus, we see that the narrator finds the display cases at her father’s ink shop to be look better than the ones from the studio at Immortal Heart village, and we should expect our answer choice to refer to this. Answer Choice A says that thinks that her family’s ink shop “displays the family’s ink more impressively.” This does match our understanding of the Text, so we should keep this option. Answer Choice B says that the family’s ink shop “is more conveniently located for the public,” which neither the Passage nor our Text mention, so we should eliminate this option. Answer Choice C says that the family’s ink shop “provides greater individual attention to customers,”

which we should also eliminate because neither our Text nor our Passage mentions it. Answer Choice D says that the family’s ink shop “offers a larger space for presenting products,” which is also incorrect because the narrator highlights the appearance of the display cases and not the size of the space. This leaves us with only Answer Choice A, which we should choose as our correct option. 7.R.11 & 8.R.11 Answer Choices C and C are the correct answers because Lines 52-55 say, “You simply write what is swimming on the top of your brain. And the top is nothing but pond scum, dead leaves, and mosquito spawn,” which best answers how “Precious Auntie would consider a hastily written first draft of a story to be.” For Question 7, Answer Choice C most closely matches this Text when it says Precious Auntie would consider such a story, “essentially worthless in and of itself.” For Question 8, Answer Choice A is incorrect because it only tells us that Precious Auntie believes everything has a purpose, not how she would consider a hastily written first draft. Answer Choice B is incorrect because it only comments on the qualities of “good ink,” not how she would consider a first draft. Answer Choice D is incorrect because it details the importance, thought process and benefit of using an inkstick and inkstone, not how she would consider a first draft. How to solve this? Question 7 asks, “Based on the artistic philosophy expressed in the fourth paragraph (lines 46-59), it is reasonable to infer that Precious Auntie would consider a hastily written first draft of a story to be [...]?” Because this is a Paired Question, we should begin by going through the answer choices for Question 8 and looking for any texts that can answer how Precious Auntie would consider such a draft. Our attention should be drawn especially to the words “hastily written,” which tell us that our answer will probably have something to do with a hurried, poorly completed work of art. For Question 8, Answer Choice A says, “As he said this, Precious Auntie came back into mind. I was remembering how she taught me that everything, even ink, had a purpose and a meaning.” Because this answer choice doesn’t tell us anything about how Precious Auntie would consider a “hastily, written first draft,” we should eliminate this option. Answer Choice B says, “Good ink cannot be the quick kind, ready to pour out of a bottle.” Because this answer choice only comments on the quality of ink and not a quickly written draft, we should eliminate this option. Answer Choice C says, “You simply write what is swimming on the top of your brain. And the top is nothing but pond scum, dead leaves, and mosquito spawn.” Expanding the reference a little, we see that here Precious Auntie is discussing the “quick kind” of “modern ink,” and contrasting it the slower kind of ink. Because these lines reference a written first draft as well as the speed with which it’s written, we should keep this option. Answer Choice D says, “You push and you ask yourself, What are my intentions? What is in my heart that matches my mind?” Because this answer choice refers to the thoughts that follow the use of an old version of ink, involving the inkstick and inkstone, and the slowness of that ink, we should eliminate this option. That leaves us with Answer Choice C, which we should for our correct answer for Question 8.

Answer Choice C says, “You simply write what is swimming on the top of your brain. And the top is nothing but pond scum, dead leaves, and mosquito spawn.” Taking this as our evidence for how Precious Auntie would feel about a “hastily written first draft,” we can say that she would take a strongly negative stance towards one, considering it “pond scum, dead leaves and mosquito spawn.” For Question 7, Answer Choice A says that Precious Auntie would consider a draft like this to be, “emotionally raw and powerful.” The word “raw” might be the only plausible thing about this answer choice, but because the Text doesn’t tell us anything about a draft being emotional and powerful, and because it isn’t negative enough, we should eliminate this option. Answer Choice B says that she would consider such a draft to be, “creatively satisfying for the author.” This answer choice is also not negative enough, nor does our Text talk about the creative satisfaction of the author, so we should eliminate this option as well. Answer Choice C says she would consider such a draft to be “essentially worthless in and of itself.” Because this answer choice is both negative and captures the Precious Auntie’s view of the work as “worthless” or “nothing but pond scum,” we should keep this option. Answer Choice D says that Precious Auntie would consider such a draft to be “inappropriately analytical for a piece of art.” Because our Text doesn’t mention the analytical nature of such a draft, and because this option isn’t negative enough, we should eliminate it. This leaves us with answer Choice C, which we should choose as our correct answer for Question 8. 9.R.11 Answer Choice B is the correct answer because the word “matches” in Line 59 can be best replaced by “corresponds with,” emphasizing the [...] as opposed to [...].

● What is in my heart that matches my mind? ● What is in my heart that corresponds with my mind?

Answer Choice A is incorrect, because the line isn’t emphasizing that the heart and mind are competing with each other.

● What is in my heart that competes against my mind? Answer Choice B is incorrect, because the line isn’t emphasizing that [...]

● What is in my heart that runs counter to my mind? ● What is in my heart that treats equally my mind?

10.R.11 Answer Choice C is correct because Line 68 emphasizes the quality of the ring of the bell, and especially the purity and distinctness of the tone, making the word “clean” best replaced by the word “distinct.”

● And I heard a sound as clean and pure as a small silver bell. ● And I heard a sound as distinct and pure as a small silver bell.

Answer Choice A is incorrect because it doesn’t make sense to describe the sound of the bell as “complete,” as opposed to “incomplete.” Answer Choice B is incorrect because it also doesn’t make sense to call the sound of the bell “skillful.” Answer Choice D is incorrect because the line isn’t emphasizing the “upright” or “proper” sound of the bell.

Reading 2

Passage

● a study show how technology changes the way our memories function ● it shows how people remember fewer facts and less information ● internet has become part of our “transactive memory” source and part of daily habits ● the four experiments in the study ● questions about whether or not this is negative ● how memory has changed what we remember, and future studies of interdependence

between computers and humans 11.R.11 (hard) Answer Choice D is the correct answer because the Passage can best be described as tracking a particular study that examines how the Internet affects memory, as the title itself suggests, “How the Web Affects Memory.” Answer Choice A is incorrect because the Passage looks at memory and not “critical thinking.” Answer Choice B is incorrect because, although the Passage does suggest some concern over the negative effects of humans’ dependence on computers, it never makes so strong a claim as to assert that they have become “overly dependent.” Answer Choice C is incorrect because the Passage doesn’t give enough evidence to support to strong claim that people’s capacity for memory is “much weaker” than it used to be and because it concludes with a statement that memory is changing, not necessarily becoming weaker.

How to solve this? The Questions asks, “The main purpose of the passage is to [...]?” To solve this, we should both have a general understanding of the Passage as a whole and also find specific sentences that tell us what it’s about. See [above / prior] for a general outline of the Passage. Specifically, we should begin by looking at the subtitle for the Passage which reads, “How the Web Affects Memory,” which gives us a pretty good indication of what the Passage is about. Next, we should look at the first paragraph, which introduces Wegner’s study and tells us that it is “changing the way our memories function.” Finally, we can look at the conclusion, which tells us that “even though we may not be taxing our memories to recall distinct fact, we are still using them to consider where the facts are located and how to access them.” Taking all these together, we can say that the Passage primarily concerns a single study that details how the Internet affects and changes our memories, without making strong claims about the negative nature of this impact, and should expect our answer choice to concern this. Answer Choice A says that the purpose of the passage is to “describe a series of experiments on the way technology interferes with critical thinking.” This answer choice might seem initially plausible, but because the Passage concerns the effect of technology on memory and not “critical thinking,” we should eliminate this option. Answer Choice B says that the purpose of the passage is to “assert that people have become overly dependent on computers for storing information.” This answer choice might also seem plausible because, at first, the Passage seems to suggest that technology affects our memory negatively. But, we should note both the strong and judgmental nature of the phrase “overly dependent,” and look for more direct evidence for the claim. Towards the end of the Passage we see that the author of the study finds that technology has primarily caused memory to change and “concedes that questions remain about whether dependence on computers will affect memories negatively.” Because of this, we should eliminate this answer choice. Answer Choice C says that the purpose of the passage is to “discuss the idea that humans’ capacity for memory is much weaker than it once was.” This answer choice is somewhat similar to the previous choice, in that it both presents a strong claim about the effect of technology on humans’ capacity for memory, and because it presents that effect as explicitly negative. These claims however, don’t find support in the Passage, which never claims that human memory is “much weaker” or takes a negative stance towards the effect of technology on human memory, so we should eliminate this option. Answer Choice D says that the purpose of the passage is to “share the findings of a study examining the effect of computer use on memory recall.” Because this answer choice matches our understanding of the Passage and because it presents a neutral stance towards the effect of technology on memory (simply “examining the effect”), we should keep this option. That leaves us only with Answer Choice D, which we should choose as our correct answer.

12.R.11 Answer Choice D is the correct answer because Lines 72-75 say that “even though we may not be taxing our memories to recall distinct facts, we are still using them to consider where the facts are located and how to access them,” which most clearly tells us “that reliance on computers does not necessarily diminish human memory.” Answer Choices A, B and C are incorrect because the lines all present [negative instances] of the effect of computers on memory without adding the claim that, in some cases, computers don’t diminish human memory. How to solve this? The Question asks, “Which choice best supports the idea that reliance on computers does not necessarily diminish human memory [...]?” [Because the Passage mostly takes a suggestively negative view towards the effect of technology on human memory, we should expect that any claim about computers not diminishing human memory should be presented as a qualification, or counter [Qualification Rule]]. Answer Choice A says, “But Harvard professor of psychology Daniel Wegner’s recent research proves that websites — and the Internet — are changing much more than technology itself.” Because this answer choice doesn’t tell us the [exception, qualification] of the effect of technology on memory, but only introduces the study that suggests that technology changes memory, we should eliminate this option. Answer Choice B says that, “Wegner points out that we never have to stretch our memories too far to remember the name of an obscure movie actor or the capital of Kyrgyzstan [...].” Because this answer choice only tells us a negative effect of technology on memory and not an [exception, qualification], we should eliminate this option as well. Answer Choice C says, “Students who have trouble remembering distinct facts, for example, may struggle to employ those facts in critical thinking.” This also presents a suggestively negative effect of technology on memory and not an [exception, qualification], so we should eliminate this option as well. Answer Choice D says that “even though we may not be taxing our memories to recall distinct facts, we are still using them to consider where the facts are located and how to access them.” This answer choice does give us something like an [exception, qualification], and tells us clearly that “reliance on computers does not necessarily diminish human memory,” so we should choose keep this option and choose Answer Choice D as our correct answer.

13.R.11 Answer Choice D is the correct answer because Lines 19-21 introduce the concept of “transactive memory” and give the example, “as when a husband relies on his wife to remember a relative’s birthday.” This most closely matches Answer Choice D which says, “illustrate the concept of a transactive memory source using a familiar situation.” Answer Choice A is incorrect because the lines don’t tell us that “people who are closely related tend to have shared memories.” Answer Choice B is incorrect because the lines don’t talk about the development of external source sof memory. And Answer Choice C is incorrect because the lines give an example of transactive memory sources, and don’t emphasize the effectiveness and accuracy of these sources. How to solve this? The Question asks, “In context, the reference to remembering a relative’s birthday mainly serves to [...]?” To solve this, we should go back to the mention in the Passage of remembering a relative’s birthday and look at its function in its surrounding context. To find this Text, we can use the [Question Order Rule] and predict that it will come earlier in the Passage and probably before Line 26. Moreover we can scan for the words “relative” and “birthday.” In Lines 19-21 we find, “First hypothesized by Wegner in 1985, transactive memory exists in many forms, as when a husband relies on his wife to remember a relative’s birthday.” The introduction of this mention of remembering a relative’s birthday, beginning with “as,” shows that the mention is used as an example of “transactive memory,” and we should expect our answer choice to relate to this. Answer Choice A says that the reference serves to “show that people who are closely related tend to have shared memories.” Because this doesn't’ capture our understanding of the Text, which doesn’t say that relatives have shared memories, we should eliminate this option. Answer Choice B says that the reference serves to “demonstrate how people initially developed external sources of memory,” which also doesn’t match our understanding of the Text, so we should eliminate this option. Answerr Choice C says that the reference serves to “emphasize the effectiveness and accuracy of transactive memory sources,” which also doesn’t match our understanding of the Text, so we should eliminate this choice as well. Answer Choice D says that the reference serves to “illustrate the concept of a transactive memory source using a familiar situation.” Because this does capture our understanding of the Text, that it serves as an example of the concept of a transactive memory source, we should keep this option. That leaves us only with Answer Choice D which we should choose as our answer choice.

14.R.11 a shitty fuck question and you should ignore it (no good way to decide between B and C) 15.R.11 Answer Choice B is the correct answer because Lines 25-26 tell us that computers and technology have now become parts of our memory or “extensions of” it, which most closely matches “additions to” and can be replaced by it.

● Now computers and technology as well are becoming virtual extensions of our memory. ● Now computers and technology as well are becoming virtual additions to our memory.

Answer Choice A is incorrect because the Lines don’t tell us that computers and technology delay our memory. Answer Choice C is incorrect because it’s unclear how computers and technology could become “lengths of our memory,” and doesn’t preserve the meaning we want. Answer Choice D is incorrect because the Lines don’t mean that computers and technology develop our memory, just that they’re further parts of it. 16.R.11 & 17.R.11 Answer Choices C and A are the correct answers because Lines 42-45 say that, “In the first experiment, participants demonstrated that they were more likely to think of computer terms like ‘Yahoo’ or ‘Google’ after being asked a set of difficult trivia questions.” This most closely matches Answer Choice C, which says “people are inclined to think of specific information sources” when they are “asked to provide facts that are not already familiar to them.” For Question 17, Answer Choice B is incorrect because it only provides the context for an experiment, and doesn’t show people thinking of specific information sources. Answer Choice C is incorrect because it only provides a result of the experiment involving participants remembering statements, and not thinking of a specific information source. Answer Choice D is incorrect because the mentions of ‘Yahoo’ and ‘Google’ qualify as better instances of “specific information sources” than the “folder locations” mentioned in these lines. For Question 16, Answer Choice A is incorrect because the participants think of Yahoo and Google when being asked trivia questions, not told to “memorize details that will then be made inaccessible.” Answer Choice B is incorrect, because the participants in the Texts are not “directed to develop a system for organizing and saving content.” Answer Choice D is incorrect because the participants are also not “prompted to identify terms related to dependence on computers.”

How to solve this? Question 16 asks, “The discussion of the experiments suggests that people are inclined to think of specific information sources in response to being [...]?” Because this is a Paired Question, to solve this we should go through the answer choices for Question 17, focusing on finding texts that show us people thinking of “specific information sources.” Answer Choice A says, “In the first experiment, participants demonstrated that they were more likely to think of computer terms like ‘Yahoo’ or ‘Google’ after being asked a set of difficult trivia questions.” Because this answer choice does show people thinking of specific information sources, namely Yahoo and Google, we should keep this option. Answer Choice B says, “Half the subjects were told that their work would be saved to a computer; the other half were informed that the statements would be erased.” Because this only sets up the experiment to follow and doesn’t show participants thinking of specific information sources, we should eliminate this option. Answer Choice C says that, “In subsequent memory testing, participants who were told their work would not be saved were best at recalling the statements.” Because this answer choice shows individuals recalling statements themselves and not particular memory sources, we should eliminate this option. Answer Choice D says that, “the participants proved better able to recall the folder locations than the statements themselves.” This answer choice might seem plausible, because it shows people thinking of a particular information source, that of the folders. However, because this answer choice doesn’t mention very “specific” information sources, especially compared to the Yahoo and Google of Answer Choice A, we can eliminate this option. This leaves us with Answer Choice A, which we should choose as our correct answer for Question 17. These lines say, “In the first experiment, participants demonstrated that they were more likely to think of computer terms like ‘Yahoo’ or ‘Google’ after being asked a set of difficult trivia questions.” So, we see people thinking of “specific information sources” in response to being asked a “set of difficult trivia questions,” or questions with answers unknown to the participants, so we should expect our answer choice to relate to this. For Question 16, Answer Choice A says that the experiments show people thinking of specific information sources in response to being “required to memorize details that will then be made inaccessible.” Because our Text shows participants thinking of these sources in response to being asked trivia questions and not memorizing details, we should eliminate this answer choice. Answer Choice B says that the experiments show people thinking of specific information sources in response to being “directed to develop a system for organizing and saving content.” Because our Text doesn’t show participants being asked to develop an organizing system, we should eliminate this option. Answer Choice C says that the experiments show people thinking of specific information sources in response to being “asked to provide facts that are not already familiar to them.” Because this answer choice does match our Text, and “a set of difficult trivia questions,” we should keep this option. Answer Choice D says that the experiments show people thinking of specific information sources in response to being “prompted to identify terms related to dependence on computers.” Because the participants are asked to identify terms related to computers, but difficult trivia questions, we should eliminate this option as well. That

leaves us with Answer Choice C, which we should choose as our correct answer for Question 16. 18.R.11 Answer Choice A is the correct answer because Lines 66-68 say that students who can’t remember distinct facts will have difficulty using or employing those facts in critical thinking, and because “employ” can best be replaced by “utilize.”

● Students who have trouble remembering distinct facts, for example, may struggle to employ those facts in critical thinking.

● Students who have trouble remembering distinct facts, for example, may struggle to utilize those facts in critical thinking.

Answer Choice B is incorrect because the line doesn’t mean that students will “enroll” facts into critical thinking, like into a school. Answer Choice C is incorrect because the students aren’t said to “exert” facts in critical thinking, which implies a kind of inappropriate force or struggle. Answer Choice D is incorrect because it’s unclear what it would mean for students to “assign” facts in critical thinking. 19.R.11 Answer Choice C is the correct answer because the first column in the series of columns gives data for the participants who could remember “statements and folder locations,” or “both parts of the information given to them.” This column raises to between 15 and 20%, which most closely matches Answer Choice C which says 17%. Answer Choices A, B and D are incorrect because they don’t accurately measure the first column in the graph. How to solve this? The Question asks, “According to the graph, approximately what percentage of participants remembered both parts of the information given to them during the fourth experiment?” To solve this, we should go back to graph and try to determine which part of it measures the percentage of participants who remembered both of the information given to them. Along the bottom of the graph, we notice four labels for each of the four columns: (1) remember statements and folder locations, (2) remember statements not folder locations, (3) remember folder locations not statements and (4) remember nothing. Only the first column shows those who remembered “both parts of the information given to them,” so we should find the percentage for this column. It rises to about halfway between 15 and 20%, which most closely matches Answer Choice C, which we should choose as our correct answer.

20.R.11 Answer Choice D is the correct answer because the largest single group of participants represented in the graph refer to those who “remember nothing,” and, in the discussion of the four experiments Wegner performed (Paragraph 5), there’s no mention of participants who remembered nothing. Because we have no evidence about this group, Answer Choice D, “there is not enough information to determine the cause of the results for those participants,” makes the most sense. Answer Choices A, B and C are incorrect because, while they may be plausible explanations for the group represented, we have no evidence in the Passage to support one over the other. How to solve this? The Question asks, “Based on the description of Wegner’s fourth experiment, what is the most likely explanation for the findings for the largest single group of participants represented in the graph?” To solve this, we should first go back to the graph, find the largest group represented, and then determine what the Passage says about it. Going back to the graph, we see that the column for the group who “remembered nothing” is the largest single group represented. Because the Question asks specifically about Wegner’s fourth experiment, we should go back to the Passage to the paragraph where these experiments are described (Paragraph 5). In this paragraph, however, and even the following discussion about conclusions, we see that nothing is mentioned about the group who “remembered nothing,” which means that we can’t make any claims about them and should expect our answer choice to reference this. Answer Choice A says that the participants in the group “focused on remembering the folder locations,” which is incorrect because we have no evidence to support it. Answer Choice B is incorrect because we also have no evidence that the participants in the group “attempted to remember the statements and folder locations.” Answer Choice C is also incorrect because, although it might be plausible, there’s no evidence that the participants in the group “did not attempt to remember any specific pieces of information.” Answer Choice D says that “there is not enough information to determine the cause of the results for those participants,” which most closely matches our own answer, so we should choose Answer Choice D as our correct answer for this question.

Reading 3

Passage Outline

● why guppies are good for studying evolution, and Reznick’s experiment (Paragraph One) ● two types of guppy environment: predatory and not (Paragraph Two) ● different traits between the guppies in each environment (Paragraph Three) ● the experiment to show how quickly these traits could have evolved (Paragraph Four) ● results of experiment and other mentions of quick evolution (Paragraph Five)

21.R.11 Answer Choice A is the correct answer because Lines 3-5, in the first paragraph, say that the rate of guppy reproduction “makes guppies ideally suited for studying the rate of evolution,” and because the Passage continues to discuss observational experiments of guppies. Answer Choice B is incorrect because the paragraph doesn’t mention the value of studying guppy offspring “shortly after birth,” and because the later Passage doesn’t emphasize it either. Answer Choice C is incorrect because the first paragraph doesn’t introduce a theory, and especially not one “at the center of an ongoing scientific debate.” Answer Choice D is incorrect because the first paragraph doesn’t talk about a “new field of scientific inquiry.” How to solve this? The Question asks, “The first paragraph mainly serves to [...]?” To solve this, we should have both a general understanding of the Passage and mores specific understanding of the paragraph works in its immediate context. [See above for the general outline of the Passage]. The first paragraph mentions (1) the reproductive rate of guppies, (2) why this makes them “suited for studying the rate of evolution,” and (3) Reznick’s study which makes use of this fact. Because the Passage goes on to look at an experiment performed by Reznick which relies on guppies’ rapid rate of reproduction, we can say the first paragraph serves to give us a reason why these guppies make a good topic of study, and should expect our answer choice to reference this. Answer Choice A says that the first paragraph serves to “establish the reason why a certain species was selected for scientific observation.” Because this matches our own understanding of the paragraph, that it gives us a reason why guppies were chosen for study, we should keep this option. Answer Choice B says that the first paragraph serves to “illustrate the value of studying the offspring of a particular animal shortly after birth.” This answer choice might seem plausible, because it mentions the value of studying a particular species. However, because the paragraph doesn’t mention the specific value of studying these animals shortly after birth and because the experiments don’t show Reznick doing so, we should eliminate this option. Answer Choice C says that the first paragraph serves to “introduce a theory at the center of an ongoing

scientific debate.” Because the first paragraph doesn’t introduce a theory, and especially not one at the center of an ongoing scientific debate, we should eliminate this option. Answer Choice D says that the first paragraph serves to “offer a rationale for the prevalence of a new field of scientific inquiry.” However, because the first paragraph doesn’t mention a “new field of scientific inquiry,” or its prevalence, we should eliminate this option. That leaves us with Answer Choice A, which we should choose as our correct option. 22.R.11 Answer Choice B is the correct answer because Lines 14-16 describe a “lucky guppy” as being “born above a waterfall or a set of rapids, which keep out the predatory fish called pike cichlids found in calmer downstream waters.” This most closely matches Answer Choice B, which says that a “lucky guppy” is one that “inhabits an environment that provides natural protection from predators.” Answer Choice A is incorrect, because the lines don’t establish that a guppy is lucky because it’s born in a river with an “established guppy population.” Answer Choice C is incorrect because the lines emphasize that a guppy is lucky for living apart from predators, not for managing the risks of living near a waterfall. Answer Choice D is incorrect because, while it is true that a guppy is lucky for avoiding predators, these lucky guppies live above a waterfall and not in downstream waters, where in fact more predators live. How to solve this? The Question asks, “In describing the living conditions of guppies, the author indicates that a ‘lucky guppy’ (line 14) is one that [...]?” To solve this, we should go back to Line 14 and look at its surrounding context to determine how the author describes a “lucky guppy.” Lines 14-16 say, “A lucky guppy is born above a waterfall or a set of rapids, which keep out the predatory fish called pike cichlids found in calmer downstream waters.” This line tells us clearly that a guppy is luck because it avoids predators by living above a waterfall, as opposed to downstream in calmer waters, and we should expect our answer choice to relate to this. Answer Choice A says that a lucky guppy “is born in a major river having an established guppy population.” Because our lines don’t mention the benefit of being born in such a river, we should eliminate this option. Answer Choice B says that a lucky guppy “inhabits an environment that provides natural protection from predators.” Because this answer choice does match our understanding of the Passage, we should keep this option. Answer Choice C says that a lucky guppy “manages to navigate the risks associated with living near a waterfall.” Because our Lines don’t mention guppies navigating the risks of living near a waterfall, but instead avoiding certain risks by living near one, we should eliminate this option. Answer Choice D says that a lucky guppy “avoids predatory fish by living in calmer downstream waters.” This answer choice might seem plausible because our Lines do mention avoiding predatory fish, but because these guppies avoid predatory fish by living upstream near a waterfall and not in the downstream where predators are more prevalent, we should eliminate this option. That leaves us with Answer Choice B, which we should choose as our correct answer.

23.R.11 Answer Choice D says that “Although small predatory killifish occurred in these new sites new sites, these fish do not pose anything close to the danger of the cichlids,” which most clearly tells us that “the streams used by Reznick’s team in their real-world study were not entirely free of predators.” Answer Choice A is incorrect because these lines only describe what makes a “lucky guppy,” and doesn’t discuss Reznick’s experiment or its streams still containing predators. Answer Choice B is incorrect because these linesonly describes the difference in mortality rates for two guppy populations, and not Reznick’s experiment. Answer Choice C is incorrect because these lines only describe the type of of experiment Reznick performed, and doesn’t mention that the streams he worked in still have some predators. How to solve this? The Question asks, “Which choice provides the best evidence for the conclusion that the streams used by Reznick’s team in their real-world study were not entirely free of predators?” To solve this, we should go through each of the answer choices given, looking for the one that best tells us about the streams used by Reznick. [...]. 24.R.11 Answer Choice A is the correct answer because in Lines 43-44 Reznick describes treating streams “like giant test tubes” and performing experiments in them. This most closely matches Answer Choice A which says that such streams, “provide suitable experimental conditions.” Answer Choice B is incorrect because neither the Text nor our Passage talk about such streams being used to promote “cooperative behaviors in specimens.” Answer Choice C is incorrect because the streams don’t “expedite the rate of genetic changes,” but instead provide a place to manipulate and observe nature. Answer Choice D is incorrect because the streams aren’t used to “solve widespread environmental problems.” How to solve this? The Question asks, “In lines 43-44, Reznick uses the phrase “giant test tubes” to suggest that certain streams can [...]?” To solve this, we should go to the lines referenced and understand what they say about the streams. Lines 42-46 say “Reznick realized that he could, as he put it in a 2008 paper, ‘treat streams like giant test tubes by introducing guppies or predators’ to place they had not originally occurred, and then watch as natural selection acted on the guppies.” Based on this, we see that the streams mentioned are used by Reznick as places to experiment and manipulate nature experimentally, and should expect our answer choice to refer to this. Answer Choice A says that the phrase is used to suggest that certain streams can “provide suitable experimental conditions.” Because this answer choice does match our understanding of the Text, we should keep this option. Answer Choice B says that the phrase is used to suggest that certain streams can “promote cooperative behaviors in specimens.” Because the lines mentioned don’t discuss how streams promote cooperative behavior, we should eliminate this option. Answer Choice C says that the phrase is used to suggest that certain streams can

“expedite the rate of genetic changes.” Because the lines don’t mention that streams expedite this rate of change, we should also eliminate this option. Answer Choice D says that the phrase is used to suggest that certain streams can “solve widespread environmental problems.” Because the lines don’t say that streams can affects environmental problems, we should also eliminate this option. That leaves us with Answer Choice A, which we should choose as our correct answer. 25.R.11 Answer Choice C is the correct answer, because Lines 46-52 tell us that the “real-world manipulation of nature,” called “experimental evolution,” has become increasingly popular, or commonly used, among scientists. “Widespread” can most nearly replaces the word “popular” here.

● This kind of real-world manipulating of nature is called ‘experimental evolution,’ and it is growing increasingly popular among scientists [...].

● This kind of real-world manipulating of nature is called ‘experimental evolution,’ and it is growing increasingly widespread among scientists [...].

Answer Choice A is incorrect because the Lines don’t emphasize that the method of “experimental evolution” is now “accessible” to scientists, whereas before it couldn’t be used. Answer Choice B is incorrect because the Lines don’t emphasize that the method of “experimental evolution” is growing increasingly “suitable” or workable, whereas before it didn’t work as well. Answer Choice D is incorrect because the Lines don’t emphasize that the method is increasingly “likable,” whereas before it was unliked. 26.R.11 (hard) Answer Choice B is the correct answer because Reznick’s findings depend on the environment in which he releases the guppies containing little to no predators, so that if “the new site into which Reznick released the guppies is inhabited by fish that are found to be as predatory as the cichlids in the original sites,” this would undermine his finding that the lack of predators in the experimental site influenced the rapid evolution of new traits. Answer Choice A is incorrect because Reznick’s findings don’t depend on a uniform rate of genetic-shift for all guppies worldwide. Answer Choice C is incorrect because, although finding that experimental evolution is harmful to an environment might be concerning, it wouldn’t undermine Reznick’s findings which don’t depend on experimental evolution not being harmful to an environment. Answer Choice D is incorrect because if the transplanted guppies were found to mature later than the guppies that live downstream, this would reinforce not undermine Reznick’s findings. How to solve this? The Question asks, “Which finding, if accurate, would undermine Reznick’s findings?” To solve this, we should first go back to the part of the Passage that most specifically mentions Reznick’s findings, and determine the basis on which they rest. Based on that we can

determine what finding would most likely undermine his findings. Because this question focuses on Reznick’s findings or conclusions, we should focus on the later part of the Passage, that deals with his conclusions regarding the experiments. Moreover, we can use the [Question Order Rule] to predict that the most relevant Text for this question will come after Line 49. Paragraph 5, from Lines 53 to 72, most directly deals with Reznick’s findings, which show that when he takes fish from predatory environments and places them in non-predatory environments, they show a rapid shift in towards traits similar to guppies which exist in naturally non-predatory environments. Anything that would disrupt or disprove part of the logical foundation of his findings (the fish not actually showing any change, fish randomly changing in other environments, a confounding variable, etc.) could be our correct answer. Answer Choice A says that it would undermine Reznick’s findings if “guppies examined in other parts of the globe exhibit genetic shifts in traits at a different rate from that exhibited by the guppies Reznick examined.” Because his findings don’t depend on the uniformity of the rate of genetic shifts across the globe, we should eliminate this option. Answer Choice B says that it would undermine Reznick’s findings if “the new site into which Reznick released the guppies is inhabited by fish that are found to be as predatory as the cichlids in the original sites.” Because Reznick’s findings do depend on the fact that his new site contains no predators and because he couldn’t then conclude that the change in traits exhibited by the transplanted guppies was caused by a predator free environment, we should keep this option. Answer Choice C says that it would undermine Reznick’s findings if “experimental evolution is shown to be harmful to the environments where studies like Reznick’s are conducted.” Because his findings don’t depend on evolutionary experiments not being harmful to the environments they’re performed in, they can’t undermine his findings, and we should eliminate this option. Answer Choice D says that it would undermine Reznick’s findings if “the descendants of Reznick’s transplanted fish are proven to mature later than the guppies living below the waterfall.” Because the guppies living below the waterfall live in relatively predator-free environments and mature later because of it, this finding would actually reinforce Reznick’s findings because the transplanted guppies exhibit traits like those in naturally predator-free environments, so we should eliminate this option as well. That leaves us with Answer Choice B, which we should choose as our correct option. 27.R.11 & 28.R.11 Answer Choices A and D are the correct answers because Lines 67-72 say that, “other studies of guppies in Trinidad have shown evolutionary change in as few as two and a half years [...],” which most clearly matches Answer Choice A, that “some genetic traits will evolve more readily than others.” For Question 28, Answer Choices A, B and C are all incorrect because none of the the choices tell us about experiments in Trinidad, but only the methodology and set up for Reznick’s experiment. For Question 27, Answer Choices B, C and D are all incorrect because our lines only mention the rapid change of shown in some experiments with guppies, and not the relative dangers of predatory fish, how guppies fare in different environments or the differences in preventing genetic shifts.

How to solve this? Question 27 asks, “It can most reasonably be inferred from the passage that the experiments in Trinidad have shown which of the following about guppies?” Because this is a Paired Question, to solve it we should go through the answer choices for Question 28, looking for any texts that tell us about the experiments in Trinidad. Using the [Question Order Rule], we should also expect that our Texts will probably come towards the end of the Passage, and most likely in the final paragraph. For Question 28, Answer Choice A says, “How quickly, though, could these differences in how the two kinds of guppies lived their lives have evolved?” Because this only asks a question that sets up the experiment that Reznick performed, and doesn’t tell us anything about the experiments in Trinidad, we should eliminate this option. Answer Choice B describes the tributaries in Trinidad, and the fact that Reznick could treat them as “giant test tubes.” This answer choice does tell us something about Reznick’s experiment in Trinidad, but it’s too preliminary, only describing his reasons for choosing the location, and only sets up his experiment without giving some conclusion for multiple experiments. So, we should eliminate this option as well. Answer Choice C says that, “Along with his students and colleagues, Reznick removed groups of guppies from their predator-ridden lives below the waterfall and released them into previously guppy-free streams above the falls.” This answer choice, like the one before it, only gives us the set up for Reznick’s experiment and not any conclusion from multiple experiments, so we should eliminate this option as well. Answer Choice D says that, “Other studies of guppies in Trinidad have shown evolutionary change in as few as two and a half years, or a little over four generations, with more time required for genetic shifts in traits such as the ability to form schools and less time for changes in the colorful spots and stripes on a male’s body.” Because this answer choice does tell us about other experiments in Trinidad, and gives us some conclusions based on them, we should keep this option. This leaves only Answer Choice D which we should choose as our correct answer. Lines 67-72 tell us that other experiments in Trinidad have shown how quickly evolutionary change can occur in a group of guppies, and more time can be required for some traits than for others, so we should expect our answer choice to refer to this. Answer Choice A says that, “some genetic traits will evolve more readily than others.” Because this answer choice does match our understanding of the Text, that traits can evolve quickly and at different speeds than others, we should keep this option. [...]

29.R.11 Answer Choice D is the correct answer because the first column in the figure represents the “guppies living in the south slope high-predation environment,” and rises to about 6.5 on the graph, which most closely matches Answer Choice D which says “6 and 7.” Answer Choices A, B and C are incorrect because they don’t closely match the level of the first column. How to solve this? The Question asks, “According to figure 1, guppies living in the south slope high-predation environment produced a mean number of offspring between [...]?” To solve this, we should go back to figure in question and locate the column that measures the mean number of offspring for “guppies living in the south slope high-predation environment.” Only the first column matches this, so we should refer to it and notice that it rises to a mean of about 6.5, which most closely matches Answer Choice D. Because none of the other answer choices are as close, we should eliminate them and choose Answer Choice D as our correct answer choice. 30.R.11 Answer Choice C is the correct answer because it says, “the predation level observed in each environment had more of an effect on mean embryo mass than did slope locations.” This most closely matches our figure which shows little to no variation between different slopes with the same level of predation (see Column 1 and Column 3), but does show a lot of variation between high-predation and low-predation (see Column 1 and Column 2). Answer Choice A is incorrect because the slope location isn’t as good an indicator of the mean embryo mass as the predation level, since little variation occurs between different slope locations with the same level of predation, but does occur between different predation levels with the same slope location. Answer Choice B is incorrect is incorrect because the columns that measure the north slope location exceed that of the south slope location, meaning their mean would be higher not smaller. Answer Choice D is incorrect because the guppies born in low-predation environments actually had a greater mean embryo mass than those born in high-predation environments. How to solve this? The Question asks, “Which conclusion about the mean mass of guppy embryos is best supported by figure 2?” To solve this, we should go through each of the answer choices, noting which statements could plausibly describe the graph, and eliminating the others. Because only Answer Choice C matches the graph, we should keep that option. Because Answer Choices A, B and D don’t describe the graph accurately, we should eliminate those options and choose Answer Choice C as our correct answer.

31.R.11 Answer Choice B is the correct answer because Figure 1 shows guppies in low-predation environments tending to have fewer offspring and Figure 2 shows guppies in low-predation environments tending to have a greater mean embryo mass. That most closely matches the description in Answer Choice B, which says guppies from low-predation environments are likely to “be part of a smaller litter and have a greater mean embryo mass.” Answer Choice A is incorrect because, while the first part is true, the rate of maturity is not measured by either figure and therefore can’t be our answer. Answer Choice C is incorrect because neither figure measures the rate of survival and because guppies from low-predation environments have greater, not lesser embryo mass. Answer Choice D is incorrect because, while these guppies do have a greater mean embryo mass, they produce a lesser not greater number of offspring. How to solve this? The Question asks, “The data presented in figures 1 and 2 best support the conclusion that compared with guppies from high-predation environments, guppies from low-predation environments were more likely to [...]?” To solve this, we should begin by going back to both figures and noticing any trends that either shows for guppies born in low-predation environments. Figure 1, which measures the mean number of offspring for guppies, shows that those born in low-predation environments tend to have a much lower number of guppy-offspring than those born in high-predation environments. Figure 2 measures the mean embryo mass of guppy offspring, and show that those in low-predation environments tend to have a much greater mean embryo mass than those born in high-predation environments. So, we can say that both figures show that guppies born in low-predation environments to have a smaller mean number of offspring and a greater mean embryo mass, and we should expect our answer choice to relate to this. Because only Answer Choice B matches both of these, “smaller litter and greater mean embryo mass,” we should choose this option. We should be careful with Answer Choice A which more clearly than Answer Choice B says that these guppies “have fewer offspring,” but also measures maturity, something our figures don’t. Answer Choices C and D are incorrect because they misrepresent the information given in the figures.

Reading 4

Passage Outline

● told that the question of slavery is a political question, inappropriate for women to ask. but it is also a question of humanity and morality. (paragraph one).

● shouldn’t women be allowed to participate in this debate. is it really only a “political question”? (paragraph two).

● given that the constitution would require the north to put down a slave insurrection, and cost the lives of men, shouldn’t this question concern women? a woman’s duty to herself, family and country should be enough to participate in the debate. (paragraph three).

● even if women do admit it is a political question, so what? shouldn’t they participate in such an important debate? their country matters as much to them as anyone.

32.R.11 (4) Answer Choice D is the correct answer, because throughout the Passage, the author argues the right of women to participate in the cause of abolition and defends that right against various accusations. In Lines 51-57, she writes, “Surely this consideration is of itself sufficient to arouse the slumbering energies of women [...]; and she will not be deterred from the performance of her duty to herself, her family, and her country [...].” Or, in Lines 73-77 she says that ”the events of the last two years have cast their dark shadows before [...], shrouding the destinies of our country in more than midnight gloom, and we cannot remain inactive.” This most closely matches Answer Choice D, which says the purpose of the passage is to “encourage women to see their participation in the abolitionist cause as just and important.” Answer Choice A is incorrect because the author doesn’t accuse other abolitionists in the Passage, nor does she spend time defending the contributions of women, instead spending time justifying their position. Answer Choice B is incorrect because, while the author does defend and treat positively the cause of abolition and women’s participation in it, she never directly argues that these are explicit continuations of the spirit of the American Revolution, a very specific claim. Answer Choice C is incorrect because, while it presents the author as being sympathetic towards the cause of abolition, in the Passage the author never claims that women’s rights are meaningless while slavery exists, which is a strong and specific claim. How to solve this? The Question asks, “Smith’s main purpose in the passage is to [...]?” Because this is a [Big Picture] Question, to solve it we should go back to the Passage to get both a general understanding of its structure and content as well as to find specific texts within it that will tell us about its main purpose. [See above for general purpose]. Specifically, we can begin by looking at the subtitle, introductory paragraph and conclusion, while also scanning for any [indicators] like “but” or “however.” The subtitle for this Passage doesn’t tell us much related to its purpose. The introduction shows the author arguing against the idea that women can’t participative in the discussion of the topic of slavery because it is a “political question,” arguing

in fact that it is more than simply that. There are no strong claims in this introduction beyond the general idea that the author believes women should participate in the discussion on slavery. The conclusion begins with a series of questions, essentially making the point that even if women admit the topic to be a political one, they should be allowed to participate in it. It concludes with her stating that the country is as dear to women as anyone, and that women’s course, should still be “onward.” There’s no strong single statement in this conclusion either, but a general reinforcement of the author’s view that the topic of slavery is an important one in which women should be allowed to participate. So, we may not know in advance what our answer choice will look like, we can expect it to have something to do with the importance of women participating in the cause of abolition. Answer Choice A says that the main purpose of the passage is to “accuse fellow abolitionists of overlooking the contributions that women have made to the movement.” Although this answer choice seems favorable towards women involved in abolition, its incorrect because the author never accuses fellow abolitionists nor does she highlight specific contributions from women in the movement. Answer Choice B says that the main purpose of the passage is to “argue that the causes of abolition and women’s rights are continuations of the spirit of the American Revolution.” This answer choice also seems plausible, since it relates women’s rights and abolition in a positive manner. It’s incorrect, however, because it makes the specific claim that these causes and rights are “continuations of the spirit of the American Revolution,” a point the author never makes explicitly. So, we should eliminate this option as well. Answer Choice C says that the main purpose of the passage is to “make the case that women’s rights are meaningless while slavery exists.” This answer choice is also sympathetic to the cause of abolition, but it fails because it makes the too strong and specific claim that women’s rights are “meaningless” while slavery exists, a point the author never clearly makes. Answer Choice D says that the main purpose of the passage is to “encourage women to see their participation in the abolitionist cause as just and important.” This answer choice has the virtue of being very general, and avoiding strong or specific claims. It also matches our understanding of the Passage, that the author argues that the participation of women in the cause of abolition is “just and important.” So, we should keep this option and choose Answer Choice D as our correct answer choice.

33.R.11 (3) Answer Choice A is the correct answer because throughout the Passage, Smith asks a variety of rhetorical questions (“Will it be easy to convince them that it is no concern of theirs, that their homes are rendered desolate, and their habitations the abodes of wretchedness?” or “May we not permit a thought to stray beyond the narrow limits of our own family circle, and of the present hour?”), all of which the reader understands should be answered with “No.” This most closely matches Answer Choice A, which says that one technique Smith uses to advance her point is by presenting “claims in the form of rhetorical questions that mostly have implicit negative answers.” Answer Choice B is incorrect because the author never quotes any of her opponents. Answer Choice C is incorrect because the author doesn’t use any anecdotes or stories in the Passage. Answer Choice D is incorrect because, although the author certainly believes in her own claims, she never presents them as though they are universally held. In fact, she herself provides the claims of those who disagree with her. How to solve this? The Question asks, “Which statement provides the best description of a technique that Smith uses throughout the passage to advance her main point?” To solve this, we should rely on both memory and going back to the Passage to find what [rhetorical device / literary technique] the author uses. A common technique used in SAT Passages, and one which should stand out to us in particular is the use of rhetorical questions, which appear frequently in this Passage. We can’t be sure that these will be a part of the correct answer choice, but we should have a reasonably high expectation that they will be. Answer Choice A says that Smith uses the technique of presenting “her claims in the form of rhetorical questions that mostly have implicit negative answers.” This does match our prediction and understanding of the Passage, and, going back, we can check that the rhetorical questions asked do indeed have negative implicit answers and assure ourselves that they do, so we should keep this option. Answer Choice B says that Smith uses the technique of criticizing “her opponents by quoting self-contradictory remarks they have made.” Because nowhere in the Passage does the author criticize opponents directly, especially by quoting their self-contradictory remarks, we should eliminate this option. Answer Choice C says that Smith uses the technique of illustrating “each of her central ideas with an emotionally powerful anecdote.” Because nowhere in the Passage does the author give anecdotes or stories, we should eliminate this option as well. Answer Choice D says that Smith uses the technique of emphasizing “the reasonableness of her views by presenting them as though they are universally held.” Because the author doesn’t present her views this way, and, in fact, gives voice to those who do disagree with her, we should eliminate this option. That leaves us with Answer Choice A, which we should choose as our correct answer choice.

34.R.11 (4) Answer Choice B is the correct answer because Smith discusses the topic of slavery as a “political question” at two points in the Passage. In the first, she says that “it is not true that it is merely a political question, it is likewise a question of justice, of humanity, of morality, of religion.” Here, she expands the scope of the question of slavery beyond that of a “political question” only. In the second place, she asks, “But admitting it to be a political question, have we no interest in the welfare of our country?” and makes her point that even if slavery is indeed a “political question,” it still has relevance to women as such. These two instances in the development of her point most closely match Answer Choice B, which says that “she dismisses the designation as too narrow but then demonstrates its relevance to her audience.” Answer Choice is incorrect because the author does dismiss the designation of “political question” in the beginning of the Passage, but never by explicitly calling it “outdated,” a very specific claim. She also doesn’t later present alternative definitions. Answer Choice C is incorrect because the author never calls the designation of “political question” trite, or cliche, but instead argues that its insufficient. She also never clearly asks her audience itself to revitalize the term. Answer Choice D is incorrect because she doesn’t present the meaning the designation has for men in the beginning of the Passage. How to solve this? The Question asks, “How does Smith develop her argument about slavery as a ‘political question’ (line 3) over the course of the passage?” To solve this, we can approach the question in a number of ways. [Because this question requires a lot of back and forth, a linear approach won’t represent all of the correct steps perfectly, but should give a good enough approximation of some of the steps in the correct thought process and some of the things you should notice]. First, we can notice how each of the answer choices is split into two parts, showing us that the test understands the development of the author’s argument about slavery as a “political question” to have two parts. This means that we should go back to the Passage to look at these two specific moments, and then concentrate on proving or disproving either moment in the answer choices. In Lines 4-11, the author says of slavery that “it is not true that it is merely a political question, it is likewise a question of justice, of humanity, of morality, of religion [...].” This tells us that the author wants to expand the issue of slavery to concern more than just politics. In Lines 58-59, she returns to the topic by asking, “But admitting it to be a political question, have we no interest in the welfare of our country?” and continues to make the case that even as a “political question,” the topic concerns women. This tells us that she later embraces that designation, and we should expect our answer choice to relate to this and the first point. Answer Choice A says of Smith’s argument over slavery as a “political question” that she “claims the designation is an outdated one and then offers alternative definitions.” This answer choice might seem plausible, since it presents the sort of negative-positive structure that we saw in the Passage. But, more specifically, we should note that the author doesn’t claim that the designation of “political question” is an outdated one, but that its insufficient. Answer Choice B says of Smith’s argument over slavery as a “political question” that she “dismisses the

designation as too narrow but then demonstrates its relevance to her audience.” This answer choice does match our understanding of the Passage, where the author first states that there is more to question of slavery than it being a “political question” and then makes the point of its importance, so we should keep this option. Answer Choice C says of Smith’s argument over slavery as a “political question” that she “contends that the designation has become trite and then invites her audience to revitalize it.” This answer choice also captures the basic negative-positive structure we see in the Passage, but it makes the too-specific claim that Smith claims the designation is trite, or cliche, which she doesn’t do. Moreover, it says that Smith invites her audience to revitalize the claim, which she also doesn’t do, so we should eliminate this option as well. Answer Choice D says of Smith’s argument over slavery as a “political question” that she “describes the meaning the designation has for men and then challenges women to embrace it.” Because Smith doesn’t describe the meaning that the designation has for men in the beginning of the Passage, we should eliminate this option as well. That leaves us with Answer Choice A, which we should choose as our correct answer for the question. 35.R.11 Answer Choice B is the correct answer because, in the first paragraph, the author says of the topic of slavery that “it is not true that it is merely a political question, it is likewise a question of justice, of humanity, of morality, of religion [...],” and continues to elaborate on the reason why this is, concluding that ‘these considerations (of much more than politics) are all involved in the question of liberty and slavery.” This most closely matches Answer Choice B, which says that “Smith rejects a claim and elaborates on her reasons for doing so.” Answer Choice A is incorrect because Smith doesn’t support any conventional viewpoint, but, in fact, argues against one. Answer Choice C is incorrect because Smith doesn’t provide any historical background for her subject. Answer Choice D is incorrect because Smith doesn’t present any specific problem and then offer solutions, but rather argues about the designation of “political question.” How to solve this? The Question asks, “Which choice best summarizes the first paragraph?” To solve this, we should go back to the first paragraph and try to identify its basic structure and content. Going back, we should note that we can split the question into two basic parts:

● the subject of slavery is not merely a “political question” ● it’s also a question “of justice, of humanity, of morality, of religion,” and these and more

considerations are “all involved in the question of liberty or slavery.” Answer Choice A says that, in the first paragraph, “Smith explains a conventional viewpoint and presents evidence supporting it.” Because Smith seems to take the opposite of a conventional viewpoint, arguing against the idea that slavery is merely a “political question,” we should eliminate this option. Answer Choice B says that, in the first paragraph, “Smith rejects a claim and elaborates on her reasons for doing so.” Because this answer choice does match our understanding of the Passage, that Smith rejects the designation of “political question” and then argues why, we should keep this option. Answer Choice C says that Smith “introduces her

subject and provides historical background for understanding it.” Because nowhere in the first paragraph does Smith give historical background for anything, we should eliminate this option. Answer Choice D says that, in the first paragraph, “Smith identifies a problem and proposes steps to remedy it.” Because Smith doesn’t present her topic as a problem, and doesn’t proceed to give steps to remedy it later on, we should eliminate this option as well. That leaves us with Answer Choice B, which we should choose as our correct answer. 36.R.11 & 37.R.11 Answer Choices A and A are the correct answers because Lines 26-33 ask, “May not the ‘ornament of a meek and quiet spirit’ exist with an upright mind and enlightened intellect, and must women necessarily be less gentle because her heart is open to the claims of humanity, or less modest because she feels for the degradation of her enslaved sisters, and would stretch forth her hand for their rescue?” Here, Smith makes the case that woman can still be “gentle” and also “stretch forth her hand for their rescue.” This most closely matches Answer Choice A for Question 36, which says that it is possible for women to act “according to humanitarian principles while preserving their femininity.” For Question 37, Answer Choice B is incorrect because it only highlights that women would be affected by a war over slavery, but doesn’t claim that women are able to participate in two apparently contradictory activities. Answer Choice C is incorrect because it only claims that considering their homes, habitats and family member is itself sufficient to bring women to action, without presenting the possibility of woman performing two seemingly contradictory activities. Answer Choice D is incorrect because it only makes the claim that women feel that their country is as dear to them as anyone else. For Question 36, Answer Choice B is incorrect because our Text doesn’t present a reconciliation between adhering to a personal morality while still being neutral politically. Answer Choice C is incorrect because our Text doesn’t talk about women contributing to their family’s financial security. Answer Choice D is incorrect because neither our Text nor our Passage talk about women resisting calls for war. How to solve this? Question 36 asks, “In the passage, Smith argues that it is possible for women to engage in which activity?” Because this is a Paired Question, to solve it we should go through the answer choices for Question 37, looking for any texts that tell us which activities it is possible for women to engage in. Using the Question Order Rule, we can predict that it’s more likely for our Text to come between Lines 3 and 40. [Some alert. This Question requires some back and forth to find the right answer. After first looking at the Text, we should note that no answer choice clearly tells us about some activity that it is possible for women to do. However, after going back to the answer choices for Question 36, we can start to see that they all take the structure of bringing together two possibly contradictory activities, and that the majority have to do with tensions between domestic activities. This helps calibrate our second search, which will really help us arrive at Answer Choice A.] For Question 37, Answer Choice A says, “May not the ‘ornament of a meek and quiet spirit’ exist with an upright mind and enlightened intellect, and must women necessarily be less gentle

because her heart is open to the claims of humanity, or less modest because she feels for the degradation of her enslaved sisters, and would stretch forth her hand for their rescue?” Because this answer choice does reconcile two possibly contradictory activities for women, we should keep this option. [None of the other answer choices do.] [For Question 36, none of the other answer choices work except for Answer Choice A, where femininity and “gentleness” are matched, while acting according to humanitarian principles is matched with her stretching “forth her hand for their rescue.”] 38.R.11 Answer Choice C is the correct answer because Lines 34-42 say that, “by the Constitution of the United States, the whole physical power of the North is pledged for the suppression of domestic insurrections, and should the slaves, maddened by oppression, endeavor to shake off the yoke of the taskmaster, the men of the North are bound to make common cause with the tyrant, and put down, at the point of the bayonet, every effort on the part of the slave, for the attainment of his freedom.” This most clearly tell us that, if slaves were to revolt, the US Constitution would require the Northern States to side with the slave states and put down the insurrection. Answer Choice C most clearly matches this when it says that, “the Northern states would have to help the slave states fight the slaves’ rebellion.” Answer Choice A, B and D are incorrect because our Lines only tell us that the US Constitution would require Northern free states to “put down, at the point of the bayonet, every effort on the part of the slave,” not that they would have to “sever ties,” “give shelter to refugees,” or “provide financial assistance to rebelling slaves.” How to solve this? The Question asks, “According to Smith, the US Constitution requires which action on the part of the Northern free states if slaves were to revolt?” To solve this, we should go back to the Passage and the find the Text which can answer this. Using the [Question Order Rule], we can predict that our Text is most likely to come before Lines 40-50 and scan for the words “US Constitution” and “Northern free states.” We should finally end up on Lines 34-42, which say that, “by the Constitution of the United States, the whole physical power of the North is pledged for the suppression of domestic insurrections, and should the slaves, maddened by oppression, endeavor to shake off the yoke of the taskmaster, the men of the North are bound to make common cause with the tyrant, and put down, at the point of the bayonet, every effort on the part of the slave, for the attainment of his freedom.” These Lines tell us that the Constitution would require Northern states to partner with the South to put down a slave insurrection in the slave states, and we should expect our answer to relate to this. Answer Choice A says that, if slaves were to revolt, “The Northern states would have to sever ties with the slave states.” Because this answer choice doesn’t match either our Text or our Passage, we should eliminate this option. Answer Choice B says that, if slaves were to revolt,

“The Northern states would have to give shelter to refugees from the slave states.” This answer choice also doesn’t match our Text, so we should eliminate this option as well. Answer Choice C says that, if slaves were to revolt, “The Northern states would have to help the slave states fight the slaves’ rebellion,” which does match our answer choice, so we should keep this option. Answer Choice D says that, if slaves were to revolt, “the Northern states would have to provide financial assistance to the rebelling slaves.” Because this answer choice doesn’t match our Text or Passage, we should eliminate this option. That leaves us with Answer Choice C, which we should choose as our correct option. 39.R.11 Answer Choice D is incorrect because in Lines 38-40 the author says that “the men of the North are bound to make common cause with the tyrant,” where “tyrant” refers to the slave states in the South. Later, in Lines 82-84, the author mentions “the foot of the tyrant,” referring again to those who own slaves. This most clearly matches Answer Choice D, which says the word “tyrant” in the passage “emphasizes the unjustness of slavery.” Answer Choice A is incorrect because the word doesn’t refer to a specific individual, but to many people. Answer Choice B is incorrect because the word refers to the slaves states, not a threat of aggression from abroad. And Answer Choice C is incorrect because the word is used to describe the slave states, not to critique the limited role for women in antislavery movements. How to solve this? The Question asks, “In context, what is the main effect of Smith’s use of the word ‘tyrant’ in lines 40 and 83?” To solve this we should go back to both lines mentioned and determine how the word is being used in context, who it refers to and what effect this may have on the Passage. Lines 38-41 tell us that “the men of the North are bound to make common cause with the tyrant, and put down, at the point of the bayonet, every effort on the part of the slave.” Within its context, the word refers to the slave states, noting them as “tyrants.” In Lines 82-84, the Passage says “that the foot of the tyrant may no longer invade the domestic sanctuary,” again referring to slave owners. Based on these two, we can say that the word “tyrant” describes the “slave states,” obviously denoting them as evil and oppressive. Answer Choice A says the word “identifies a specific individual as oppressive.” Because the word refers to a group and not a specific individual, we should eliminate this option. Answer Choice B says that the word “highlights the threat of aggression from abroad.” However, because the word refers to the domestic slave states, and not to a threat from abroad, we should eliminate this option as well. Answer Choice C says the word “critiques the limited roles for women in antislavery movements.” Because the word, however, refers to slave states and not to the limited role of women, we should eliminate this option as well. Answer Choice D, which has the virtue of generality, most clearly matches our understanding of the use of the words, so we should keep it. This leaves us with only Answer Choice D which we should choose as our correct answer for Question 39.

40.R.11 Answer Choice C is the correct answer, because the word “slumbering” in Line 52 can be best replaced by the word “dormant,” referencing the unused or unengaged energies of woman. Answer Choice A is incorrect because it emphasizes an actual physical sleepiness, and too specifically describes an actual person not a general state. Answer Choice B suffers for the same reasons. Answer Choice D misses the point, because we want to emphasize the temporary lack of engagement, not a complete unconsciousness as well as personifying the the “energies of woman” in the same way as Answer Choices A and B. This leaves us with Answer Choice C, which we should choose as our correct answer. 41.R.11 & 42.R.11 (4) Answer Choices A and C are the correct answers because Lines 68-73 describes how the US is “rendered a hissing and a reproach throughout the world, by this system which is already tolling the death-bell of her decease among the nations,” which most clearly matches Answer Choice A for Question 41 which says that slavery is “lowering the country’s reputation in the international community.” For Question 42, Answer Choice A is incorrect because, while it refers obliquely to the effect of slavery on the US, it’s less direct than C and doesn’t match any answer Choice in Question 41. Answer Choice B is incorrect because it only asks a rhetorical question about what’s appropriate for women to think. Answer Choice D is incorrect because no answer choice for Question 41 directly references it. For Question 41, Answer Choice B is incorrect because neither our Text nor the other given choices describe women disavowing their allegiance to the country. Answer Choice C is incorrect because, while one of the texts may seem to support this option, it does so weakly and hypothetically, not giving us enough support to choose this option. Answer Choice D is incorrect because neither our Text nor the other options say that the authority of the country’s government is weakened by slavery. How to solve this? Question 41 asks, “In the passage, Smith most strongly suggests that slavery affects the United States by [...]?” Because this is a Paired Question, we should begin by going through the answer choices for Question 42, looking for any texts that tell us how slavery affects the United States. For Question 42, Answer Choice A asks, “Will it be easy to convince them that it is no concern of theirs, that their homes are rendered desolate, and their habitations the abodes of wretchedness?” Although this question is only hypothetical, and doesn’t directly describe the effect of slavery on the country, it does discuss some kind of effect, that women’s “homes are rendered desolate, and their habitations the abodes of wretchedness,” so we should keep this option. Answer Choice B asks, “May we not permit a thought to stray beyond the narrow limits of our own family circle, and of the present hour?” Because this question only refers to what’s appropriate for women to consider, and not the effect of slavery on the country, we can eliminate this option. Answer Choice C asks, “Shall we silently behold the land which we love with all the heart-warm affection of children, rendered a hissing and a reproach throughout the

world, by this system which is already tolling the death-bell of her decease among the nations?” Because this answer choice references the United States (“the land which we love”), slavery (“this system”), and the effect of slavery on the US (“rendered a hissing and a reproach throughout the world”), we should consider this a strong option and keep it as an answer choice. Answer Choice D ssays, “the events of the last two years have cast their dark shadows before, overclouding the bright prospects of the future, and shrouding the destinies of our country in more than midnight gloom, and we cannot remain inactive.” Because this answer choice also tells us about the effect of slavery on the United States, we should keep this option as well. At this point, then, we’re unable to decide between the answer choices for Question 42 and have to use Question 41 to help us make a decision. Before going back, however, we should make a mental note of the meaning for the three remaining answer choices for Question 42.

● Answer Choice A — slavery could cause homes to be “rendered desolate and their habitations the abodes of wretchedness.”

● Answer Choice C — slavery causes the US to be a “hissing and a reproach throughout the world.”

● Answer Choice D — slavery causes a “dark shadow” to fall over the country, and clouds its “bright prospects” and destiny.

For Question 41, Answer Choice A says that slavery affects the United States by “lowering the country’s reputation in the international community.” Because this answer choice matches Answer Choice C for Question 42, a strong option, we should keep this answer choice. Answer Choice B says slavery affects the United States by “leading many women to disavow their allegiance to the country.” Because none of our possible texts suggest that women would disavow their loyalty, we should eliminate this option as well. Answer Choice C says slavery affects the United States by “causing violent conflicts in many areas of the country.” This answer choice could refer to Answer Choice A for Question 42, so we can keep this option. Answer Choice D says slavery affects the United States by “weakening the authority of the country’s government.” Because none of our texts say that slavery weakens the authority of the government, we should eliminate this option as well. That leaves us with a choice between Answer Choice A and C, or between Answer Choice C and A. We should focus on eliminating one option, thinking about which could have an error or insufficient evidence. Ultimately, because Lines 48-50 refer to a possible state of affairs that slavery could cause, an insurrection and subsequent conflict, and not something that’s yet actual, it’s a weaker answer choice and we should Answer Choices A and C for our correct answers.

Reading 5

Passage Outline

Passage One

● introduce the scientist Kim Lewis, who created new antibiotic with novel beneficial effects (paragraph one)

● how they approached and thought about growing these antibiotics (paragraph two) ● how they actually grew these antibiotics (paragraph three and four) ● a reason why the new antibiotic may be so effective (paragraph five)

Passage Two

● how Lewis figured out how to grow new kinds of antibiotics ● one of the antibiotics they found, and some of its positive features ● some issues and imperfections with the new antibiotic

43.R.11 Answer Choice C is the correct answer because the first paragraph says that “‘Pathogens are acquiring resistance faster than we can introduce new antibiotics,’” which introduces the problem that the Passage goes on to address. This most clearly matches Answer Choice C which says that the purpose of the Passage is to “identify a problem that the research discussed in the passage may help to address.” Answer Choice A is incorrect, because the remainder of Passage 1 doesn’t go on to support and develop the claim that “pathogens are acquiring resistance faster than new antibiotics can be introduced.” Answer Choice B is incorrect because the first paragraph doesn’t introduce a “controversy,” but only a problem to be addressed. Answer Choice D is incorrect because the first paragraph doesn’t offer any theory, and especially doesn’t offer one that is challenged by later findings. How to solve this? The Question asks, “The first paragraph of Passage 1 primarily serves to [...]?” To solve this, we should have both a general understanding of the Passage, as well as a specific understanding of the first paragraph and what role it might play in the Passage as a whole. The general outline of Passage 1 looks like: Passage One

● introduce the scientist Kim Lewis, who created new antibiotic with novel beneficial effects (paragraph one)

● how they approached and thought about growing these antibiotics (paragraph two) ● how they actually grew these antibiotics (paragraph three and four) ● a reason why the new antibiotic may be so effective (paragraph five)

Going back to the first paragraph we see it introduces a quote by Lewis, the scientist described throughout the Passage, that says, “Pathogens are acquiring resistance faster than we can introduce new antibiotics, and this is causing a human health crisis.” Taking these two things together, the general outline of the Passage and specific meaning of the paragraph, we can say that the first paragraph serves to set up or introduce the problem that motivated the creation of the drug explored in the Passage, and should expect our answer to have something to do with this. Answer Choice A says that the first paragraph serves to “present a claim that is supported and developed over the course of the passage.” Because the claim in the first paragraph isn’t something that’s developed and supported throughout the passage, but the introduction of a problem that gets solved, we should eliminate this option. Answer Choice B says that the first paragraph serves to “introduce a controversy that the study described in the passage is intended to resolve.” This answer choice might at first seem plausible, but we should note the strong claim that the first paragraph introduces a “controversy,” or subject of debate, which is too strong of a claim for the lines given, so we should eliminate this option. Answer Choice C says that the first paragraph serves to “identify a problem that the research discussed in the passage may help to address.” Because this answer choice does match our understanding of the Text and Passage, we should keep this option. Answer Choice D says that the first paragraph serves to “offer a theory that is challenged by the findings presented in the passage.” Because the first paragraph doesn’t offer a theory, and especially not one that is later challenged in the passage, we should eliminate this option. That leaves us with Answer Choice C, which we should choose as our correct option. 44.R.11 & 45.R.11 (4) Answer Choices D and A are the correct answer choices because Lines 17-21 say, “But only about one percent of these organisms can be grown in a lab. The rest in staggering numbers, have remained uncultured and of limited use to medical science, until now.” This tells us that most of the organisms that could produce antibiotics were unable to be grown in labs, until Lewis’s method was introduced. This most clearly Answer Choice D which says that the advantage of Lewis’s method is that it “allows researchers to make use of soil bacteria that they had previously been unable to exploit.” For Question 45, Answer Choice B is incorrect because, although it does provide a possible advantage that Lewis’s team exploits, no answer choice for the previous question matches it. Answer Choice C also presents a possible advantage of the method, but likewise doesn’t have an answer choice that matches it. Answer Choice D is incorrect because it talks about the advantage and effectiveness of the teixobactin, but the method of Lewis’s team. For Question 44, Answer Choice A is incorrect because the experiment doesn’t replicate specific features in “artificial soil,” but rather grows them “in their natural environment.” Answer Choice B is incorrect because neither our Text nor our Passage tell us that Lewis’s method allows microorganisms to take in more nutrients than in their natural settings. Answer Choice C is incorrect because nothing is said about Lewis’s method affecting the cell walls of bacteria rather than the protein they produce.

How to solve this? Question 44 asks, “The author of Passage 1 suggests that an advantage of the method Lewis’s team used to grow microorganisms is that it [...]?” Because this is a Paired Question, to solve it we should begin by going through the answer choices for Question 45, looking for any texts that tell us about a possible advantage to Lewis’s team’s method. For Question 45, Answer Choice A says that “only one percent of these organisms can be grown in a lab. The rest, in staggering numbers, have remained uncultured and of limited use to medical science, until now.” This answer choice doesn’t seem to tell us too directly the benefit of Lewis’s method, but because it does suggest that the method allows for new antibiotics to be grown in culture, a possible advantage, we should keep this option. Answer Choice B says, “to do this, the team designed a gadget that sandwiches a soil sample between two membranes, each perforated with pores that allow molecules like nutrients to diffuse through but don’t allow the passage of cells.” Because this answer choice tells us about a gadget that the team developed, a possible advantage, so we should keep this option. Answer Choice C says that the team use the gadget “to trick the bacteria into thinking that they are in their natural environment.” Because this answer choice elaborates on the first one, and suggests a possible advantage of the team, we should keep this option. Answer Choice D says that, “It’s likely that teixobactin is effective because of the way it targets disease: The drug breaks down bacterial cell walls by attacking the lipid molecules that the cell creates organically.” Because this answer choice refers more to an advantage of the teixobactin itself, and not the method of the team that cultured it, we should eliminate this option. This leaves us with Answer Choices A, B and C, which we have to use Question 44 to decide between. Before looking back at that question, however, we should note briefly the point each text makes:

● Answer Choice A — previously, helpful organisms were unable to be grown in labs, but now they can be.

● Answer Choice B — the team designed a new gadget which allowed nutrients to pass through a membrane but not cells.

● Answer Choice C — the team “tricked” the bacteria into thinking that they were in their natural environment.

For Question 44, Answer Choice A says that an advantage of the method used by Lewis is that it “identifies the requirements for soil bacteria to thrive and replicates those features in artificial soil.” Because this answer choice does seem to match both Answer Choices B and C for Question 45, we can keep this option. Answer Choice B says that an advantage of the method used by Lewis is that it “enables soil bacteria to take in more nutrients than they typically consume in natural settings.” Because the Passage only says that it enables soil bacteria to absorb similar amounts of nutrients to what they absorb naturally, not more, we should eliminate this option. Answer Choice C says that an advantage of the method used by Lewis is that it “directly affects the cell walls of bacteria rather than the proteins those bacteria produce.” This answer choice seems to reference Answer Choice D from Question 45, but we should

remember that Choice D referred to an advantage of teixobactin, not Lewis’s method, so we can eliminate this option. Answer Choice D says that an advantage of the method used by Lewis is that it “allows researchers to make use of soil bacteria that they had previously been unable to exploit.” Because this answer choice matches Answer Choice A, we should keep this option. That leaves us with either Answer Choice A for Question 44 and Answer Choices B or C for Question 45, or it leaves us with Answer Choice D for Question 44 and Answer Choice A for Question 45. We should notice two things. One, that Answer Choice D for Question 44 provides more general answer choice, which is usually an advantage. Two, that Answer Choice A says that the soil used by Lewis’s team is “artificial,” when elsewhere in the Passage we’re told that the bacteria are grown in their natural environment (Line 25). For that reason, we should eliminate that option, and choose Answer Choices D and A for our correct answers. 46.R.11 Answer Choice D is the correct answer because Lines 51-56 say that “Many good antibiotic families — penicillin, streptomycin, tetracycline — come from soil fungi and bacteria and it has long been suspected that, if we could grow more types of bacteria from soil — or from exotic environments, such as deep oceans — then we might find new natural antibiotics.” The phrase, “long been suspected,” makes these Lines most clearly match Answer Choice D, which says that the author of Passage 2 would agree with the statement that the development of teixobactin “confirms a long-held belief about a potential source of new antibiotics.” Answer Choice A is incorrect because the Passage says that teixobactin is ineffective against gram-negative bacteria, not effective against it. Answer Choice B is incorrect because the development of teixobactin used an unconventional method for its development. Answer Choice C is incorrect because there’s no evidence that the development of teixobactin should cast doubt on the practicality of searching new exotic environments, and, in fact, the development of teixobactin shows that there is practicality in this kind of search. How to solve this? The Question asks, “The author of Passage 2 would most likely agree with which statement about the development of teixobactin?” To solve this, we should go back to the Passage first to try and identify what we already know about the author of Passage 2’s perspective on the development of teixobactin. Because this question is the first that concerns Passage 2, and because it references the “development” of teixobactin, we can expect our Text to come earlier in the Passage. Moreover, using the Question Order Rule, we can predict that our Text is likely to come before Line 79. In the first paragraph, we see that the author makes a number of points about the development of teixobactin. One, he takes an overall positive stance towards the development, saying that, “this simple and elegant methodology is their most important finding to my mind.” Two, he mentions that “it has long been suspected” that antibiotics could have been found by cultivating different types of bacteria. At this point, we should go down to our answer choices, looking for any choice that matches our Passage.

Answer Choice A says that the author of Passage 2 would agree that the development of teixobactin “reveals that some antibiotics are effective against gram-negative bacteria.” Because later in the Passage, the author says that these antibiotics are in fact ineffective against gram-negative bacteria, we should eliminate this option. Answer Choice B says that the author of Passage 2 would agree that the development of teixobactin “shows that conventional methods can still yield new types of antibiotics.” However, because the development of teixobactin used unconventional methods, not conventional ones, we should eliminate this option. Answer Choice C says that the author of Passage 2 would agree that the development of teixobactin “casts doubt on the practicality of searching for new antibiotics in exotic environments.” Because the discovery of teixobactin would actually support the practicality of this kind of search, we should eliminate this option as well. Answer Choice D says that the author of Passage 2 would agree that the development of teixobactin “confirms a long-held belief about a potential source of new antibiotics.” Because this answer choice does match our understanding of the Passage, that “it has long been suspected” that antibiotics could be found in the manner that they were with teixobactin, we should keep this option. That leaves us with Answer Choice D, which we should choose as our correct option. 47.R.11 Answer Choice C is the correct answer because the question, “So, what are my caveats?” most nearly means, “What issues do I see with teixobactin?” “Misgivings” most clearly replaces “caveats” here, meaning some point of apprehension or reluctance. Answer Choice A is incorrect because the author doesn’t go on to point out “exceptions” to teixobactin, but things he finds issue with. Answer Choice B is incorrect because it’s unclear and unidiomatic to say “So what are my restrictions?” Answer Choice D is incorrect because the author doesn’t go on to give “explanations,” but some reasons why he doubts the effectiveness of teixobactin. 48.R.11 Answer Choice A is the correct answer because Lines 90-91 say, “That’s going to take five years and 500 million,” with “that” referring to the “long haul of clinical trials” required to make teixobactin available as a drug. Because the author uses this phrase to express the process still required to make teixobactin available, Answer Choice A most clearly expresses the purposes of the lines when it says that they “emphasize the scale of the effort needed to make teixobactin available for consumer use.” Answer Choice B is incorrect because the $500 million mentioned are an estimated amount for the cost of teixobactin development, not an amount that the government has committed to. Answer Choice C is incorrect because the $500 million hasn’t yet been spent on teixobactin. Answer Choice D is incorrect because the author doesn’t compare the amount of money needed to develop teixobactin with the amount needed to develop other antibiotics.

How to solve this? The Question asks, “In the last sentence of Passage 2, the author uses the phrase ‘five years and $500 million’ primarily to [...]?” To solve this, we should go back to the phrase mentioned and determine its meaning and purpose in context. Lines 90-93 say, “That’s going to take five years and $500 million and these are numbers we must find ways to reduce (while not compromising safety) if we’re to keep ahead of bacteria [...],” with “that” referring to the “long haul of clinical trials” necessary to bring teixobactin to market. The phrase “five years and $500 million,” then, is used to detail his claim about the length of time and cost necessary to make teixobactin viable, and we should expect our answer choice to reference this. Answer Choice A says that the author uses the phrase to “emphasize the scale of the effort needed to make teixobactin available for consumer use.” Because this answer choice does match our understanding of the Text, we should keep this option. Answer Choice B says that the author uses the phrase to “criticize the level of funding that the government has committed to teixobactin development.” Because the author doesn’t tell us that this level of money has been committed by the government to teixobactin development, but is a hypothetical cost, we should eliminate this option. Answer Choice C says that the author uses the phrase to “underscore the amount of time and money that has already been spent researching teixobactin.” Because the $500 million is an expected amount of money, and not an amount that has already been spent, we should eliminate this option. Answer Choice D says that the author uses the phrase to “compare the amount of money spent developing teixobactin with the amount spent developing other antibiotics.” Because the author doesn’t compare the cost of teixobactin development with any other antibiotic, we should eliminate this option. That leaves us with Answer Choice A, which we should choose as our correct answer. 49.R.11 Answer Choice A is the correct answer because Passage 2 can best be described as evaluating the findings from Passage 1, commenting on the positive (“this simple and elegant methodology is their most important finding to my mind”), netural (“teixobactin [...] is less exciting to my mind, though it doesn’t look bad”), and negative (“So, what are my caveats?”) aspects of the finding. Answer Choice B is incorrect because the author of Passage 2 doesn’t suggest a modification to the methodology described in Passage 1, and, in fact, simply praises it as an “important finding.” Answer Choice C is incorrect because the two Passages don’t take the structure of one presenting concepts and the other giving concrete examples, because both provide concrete examples. Answer Choice D is incorrect because it’s too strong to say that Passage 2 takes a “dismissive stance” towards the findings mentioned in Passage 1. How to solve this? The Question asks, “Which choice best describes the relationship between Passage 1 and Passage?” To solve this, we should both have a general understanding of the structure and content of each passage and then determine the relationship between them in advance. Generally, the two passages break down like this:

Passage One

● introduce the scientist Kim Lewis, who created new antibiotic with novel beneficial effects (paragraph one)

● how they approached and thought about growing these antibiotics (paragraph two) ● how they actually grew these antibiotics (paragraph three and four) ● a reason why the new antibiotic may be so effective (paragraph five)

Passage Two

● how Lewis figured out how to grow new kinds of antibiotics ● one of the antibiotics they found, and some of its positive features ● some issues and imperfections with the new antibiotic

Moreover, we can look at specific moments in either Passage that will tell us about their relationship to one another. The titles of each proves useful for these two Passages, with Passage 1 being titled “A New Antibiotic Found in Dirt Can Kill Drug-Resistant Bacteria” and Passage 2 being titled, “This New Antibiotic Is Cause for Celebration — and Caution.” Taking both the general breakdown and title of each Passage, we can say in advance that Passage 1 details the finding of a new antibiotic, in generally positive terms, while Passage 2 discusses the good and bad of these findings, in more generally skeptical terms. Answer Choice A says that “Passage 2 offers an evaluation of the significance of the research discussed in Passage 1.” Because this answer choice matches our understanding of the relationship between the two passages, and because it’s so general, we should keep this option. Answer Choice B says that “Passage 2 suggests a modification to the methodology described in Passage 1.” Because there’s no evidence that Passage 2 ever makes this suggestion, we should eliminate this option. Moreover, we see that the author of Passage 2 actually praises the methodology of Passage 1, saying that “this simple and elegant methodology is their most important finding to my mind [...].” Answer Choice C says that “Passage 2 uses concrete examples to illustrate concepts considered in Passage 1.” Because the two Passages don’t take this structure of concept and example, and because both passages contain concrete examples, we should eliminate this option. Answer Choice D says that “Passage 2 takes a dismissive stance regarding the findings mentioned in Passage 1.” This answer choice might seem plausible, because the author of Passage 2 does express some caution towards the findings of Passage 1, but “dismissive” is a strong claim with a large burden of proof. Because there’s insufficient evidence to prove that the author is dismissive, and because Answer Choice A is so general, essentially saying that “Passage 2 gives some opinions on Passage 1,” we should eliminate this option. That leaves us with Answer Choice A, which we should choose as our correct answer.

50.R.11 Answer Choice B is the correct answer because both passages point out that teixobactin is effective against infections that are particularly resistant to antibiotics. Lines 8-10 say, “In animal tests, teixobactin proved effective at killing off a wide variety of disease-causing bacteria — even those that have developed immunity to other drugs,” and Lines 74-77 say that teixobactin “killed the tuberculosis bacterium, which is important because there is a real problem with resistant tuberculosis in the developing world.” This most closely matches Answer Choice B, which says that both passages make the point that teixobactin could be useful in “combating infections that are no longer responding to treatment with other antibiotics.” Answer Choice A is incorrect because teixobactin itself would not prove useful in making the future development of antibiotics standard, even if the methodology that produced it would. Answer Choice C is incorrect because neither passage talks about teixobactin “controlling the spread of pathogenic soil fungi.” Answer Choice D is incorrect because, again, the methodology that produced teixobactin may be useful in “shaping a new method of studying the effectiveness of antibiotics,” but teixobactin itself wouldn’t be. How to solve this? The Question asks, “Both passages make the point that teixobactin could be useful in [...]?” To solve this, we should go back to both passages and look for what they say about the usefulness of teixobactin. Passage 1 talks about the usefulness of the antibiotic in a number of places. In Lines 7-10 it says that “teixobactin proved effective at killing off a wide variety of disease-causing bacteria — even those that have developed immunity to other drugs.” In Lines 39-43 it describes how teixobactin cured mice of respiratory tract infections with “no noticeable toxic effects.” And, finally, in Lines 44-50 it describes how teixobactin works differently than other antibiotics, breaking down a bacteria’s cell walls as opposed to its proteins. We don’t have to identify all of these points to answer this question correctly, but should have the general impression that Passage 1 believes teixobactin is particularly useful in the way it treats infections, either because of how it targets disease or in its ability to treat infections previously resistant to antibiotics. Passage 2 talks about the usefulness of teixobactin in its second paragraph, saying that it “killed Gram-positive bacteria” as well as “the tuberculosis bacterium, which is important because there is a real problem with resistant tuberculosis in the developing world.” Both of these Passages, then, comment on the usefulness of teixobactin in treating infection. Moreover, both mention that teixobactin is effective against particularly resistant infections, so we can predict that our answer choice may deal with this point. Answer Choice A says that teixobactin could be useful in “standardizing the future development of antibiotics produced in laboratory environments.” Because teixobactin itself wouldn’t be responsible for standardizing this future, even if the methodology that produced it would be, we can eliminate this option. Answer Choice B says that teixobactin could be useful in “combating infections that are no longer responding to treatment with other antibiotics.” Because this answer choice matches our understanding of the Texts for both passages, we should keep this option. Answer Choice C says that teixobactin could be useful in “controlling the spread of pathogenic soil fungi” which neither passage discusses, so we can eliminate this option. Answer

Choice D says that teixobactin could be useful in “shaping a new method of studying the effectiveness of antibiotics.” Because, again, this is more true of the methodology that produced the discovery of teixobactin and not the drug itself, we should eliminate this option. That leaves us with Answer Choice B, which we should choose for our correct answer. 51.R.11 & 52.R.11 Answer Choices C and D are the correct answers because Lines 80-82 say that teixobactin “doesn’t kill the Gram-negative opportunists as it is too big to cross their complex cell wall,” while in Lines 39-43 Passage 1 says that “mice infected with bacteria that cause upper respiratory tract infections [...] were treated with teixobactin, and the drug knocked out the infection with no noticeable toxic effects.” Taking these two together, we can say that because the mice in Passage 1 were successfully cured of their infection, then “their upper respiratory tract infections were likely not caused by gram-negative bacteria.” For Question 52, Answer Choice A is incorrect because it only comments on a belief about possible sources of new antibiotics, and doesn’t relate to the mice in Passage 1. Answer Choice B is incorrect because it only comments on the methodology used by Lewis and her team, not the mice in Passage 1. Answer Choice C is incorrect because it only makes a brief, general comment on teixobactin without giving any information relevant to the mice from Passage 1. For Question 51, Answer Choice A is incorrect because neither Passage makes the case that teixobactin makes mice less susceptible to future infections. Answer Choice B is incorrect because it doesn’t make sense to say that Gram-positive bacteria “enhanced the effectiveness of teixobactin,” since that bacteria is what teixobactin targets. Answer Choice D is incorrect because Passage 1 tells us that teixobactin doesn’t target the proteins of bacteria, but rather their cell walls. How to solve this? Question 51 asks, “Information in Passage 2 best supports which conclusion about the mice in the experiment described in Passage 1?” To solve this, we should first go back to Passage 1 to find the conclusion it gives about the mice. Lines 39-43 tell us that “mice infected with bacteria that cause upper respiratory tract infections [...] were treated with teixobactin, and the drug knocked out the infection with no noticeable toxic effects.” From here, because this is a Paired Question, we should go back through the answer choices for Question 52, looking for any texts that tell us about something relevant to these mice. For Question 52, Answer Choice A says, “Many good antibiotic families — penicillin, streptomycin, tetracycline — come from soil fungi and bacteria and it has long been suspected that, if we could grow more types of bacteria from soil [...] then we might find new natural antibiotics.” Because this answer choice doesn’t tell us anything relevant to the mice mentioned in Passage 1, only about the potential sources for new antibiotics, we should eliminate this option. Answer Choice B says, “This simple and elegant methodology is their most important finding to my mind, for it opens a gateway to cultivating a wealth of potentially antibiotic-producing bacteria that have never been grown before.” This answer choice also doesn’t tell us anything about the mice in Passage 1, only the usefulness of the methodology used by Lew and her team, so we should eliminate this option. Answer Choice C says, “The first

new antibiotic that they’ve found by this approach, teixobactin, from a bacterium called Eleftheria terrae, is less exciting to my mind, though it doesn’t look bad.” Because this answer choice only introduces teixobactin and makes a brief comment on it, we should consider it unlikely to be our answer choice and eliminate it. Answer Choice D says “It doesn’t kill the the Gram-negative opportunists as it is too big to cross their complex cell wall,” with “it” referring to teixobactin. At first, this answer choice might seem equally unhelpful to answering our question, in which case we can go back to Question 51 to help us adjust and find a possible match in the answer choices. Or, we could make the logical inference here that if teixobactin can’t kill Gram-negative opportunists, and it did kill the bacteria that caused the infection in the mice from Passage 1, then that infection was unlikely to be caused by Gram-negative bacteria. For Question 51, Answer Choice A says that Passage 2 supports the conclusion that “exposure to teixobactin made them less susceptible to subsequent upper respiratory tract infections.” Because neither Passage, nor any of our possible texts make the point that exposure to teixobactin prevents against future infections, we can eliminate this option. Answer Choice B says that Passage 2 supports the conclusion that “Gram-positive bacteria enhanced the effectiveness of teixobactin against their upper respiratory tract infections.” Because teixobactin targets Gram-positive bacteria, it doesn’t make sense to say that this bacteria enhances its effectiveness, so we should eliminate this option. Answer Choice C says that Passage 2 supports the conclusion that the mice’s “upper respiratory tract infections were likely not caused by gram-negative bacteria.” Because this answer choices matches well with Answer Choice D and our understanding of that Text, we should keep this option. Answer Choice D says that Passage 2 supports the conclusion that “teixobactin attacked the proteins of the bacteria that caused their upper respiratory tract infections.” Because Passage 2 tells us that teixobactin works by targeting cell walls and not cellular proteins, we should eliminate this option. That leaves us with Answer Choice C, which we should choose as our correct answer.